1
$\begingroup$

Let $\Lambda$ be the von Mangoldt function. I am interested in understanding the average $$\sum_{n=1}^x \Lambda(n)^2.$$ By partial summation and the prime number theorem one can prove that this is $$ x(\log x) -x +O\left(\frac{x}{(\log x)^A}\right)$$ for any fixed $A>0$. But my question is whether one can see this directly from the residue of some Perron style formula? Presumably the Dirichlet series of $\Lambda^2$ has a double pole at $s=1$ but I am not aware of any nice Dirichlet series for $\Lambda^2$.

Replacing $\Lambda^2(n)$ by $(\log n) \Lambda(n)$ would change almost nothing in any asymptotics and $(\log n) \Lambda(n)$ is nicer to work with as it comes up naturally as the derivative of $\zeta'/\zeta$. The equality
$$\left(\frac{\zeta'}{\zeta}\right)'= \frac{\zeta"}{\zeta} -\left(\frac{\zeta'}{\zeta}\right)^2 = \frac{\zeta"}{\zeta'} \frac{\zeta'}{\zeta} -\left(\frac{\zeta'}{\zeta}\right)^2 $$ is equivalent to $$ -\Lambda \cdot \log =f\ast \Lambda - \Lambda \ast \Lambda ,$$ where $\cdot$ is pointwise multiplication, $\ast$ is the Dirichlet convolution and $f$ is the coefficient of $-\zeta"/\zeta'$. But $f$ looks a bit mysterious!

$\endgroup$
4
  • 2
    $\begingroup$ note that since $\Lambda(n)(\log n-\Lambda(n))$ is supported on prime powers at least $2$, the difference between $\sum \Lambda(n)^2 n^{-s}$ and $\left(\frac{\zeta'}{\zeta}\right)'$ is a Dirichlet series convergent for $\Re s >1/2$ so indeed the asymptotics and the function to apply Perron too is $\left(\frac{\zeta'}{\zeta}\right)'$ $\endgroup$ Commented Oct 3 at 21:30
  • $\begingroup$ Yes indeed! But $\Lambda(n) (\log n)$ also seems a bit unnatural. In fact I need it in a different setting where partial summation does not work... $\endgroup$ Commented Oct 4 at 7:31
  • 1
    $\begingroup$ I don't see the difficulty: just displace the contour to the left for $(\zeta'/zeta)'$. The analysis is not any harder. There are contexts in which the identity you've given is the first step towards something (since $\zeta''/\zeta$ is more amenable to a sieve approach, and $\Lambda\ast \Lambda$ gives rise to a "type II" (bilinear) sum), but this is not one of them. $\endgroup$ Commented Oct 5 at 6:19
  • $\begingroup$ Have you any thoughts, Dr. Pi, on the answer posted some time ago by Steven Clark? $\endgroup$ Commented Dec 12 at 21:53

1 Answer 1

2
$\begingroup$

The asymptotic for

$$b(x)=\sum\limits_{n\le x} \Lambda(n)\, \log(n)\tag{1}$$ is $$b(x)\approx x\, (\log(x)-1)\tag{2}$$

which follows from the explicit formula

$$b_o(x)=\lim\limits_{\epsilon\to 0} \left(\frac{b(x-\epsilon)+b(x+\epsilon)}{2}\right)\\=x\, (\log(x)-1)+\frac{\pi^2}{12}-\gamma^2-2 \gamma_1+\sum\limits_{\rho} \frac{1-\rho \log(x)}{\rho^2}\, x^{\rho}\\+\sum\limits_{n=1}^{\infty} \frac{1+2 n \log(x)}{4 n^2}\, x^{-2 n}\\=x\, (\log(x)-1)+\frac{\pi^2}{12}-\gamma^2-2 \gamma_1+\sum\limits_{\rho} \frac{1-\rho \log(x)}{\rho^2}\, x^{\rho}+\frac{1}{4} \left(\text{Li}_2\left(\frac{1}{x^2}\right)-2 \log\left(1-\frac{1}{x^2}\right) \log(x)\right),\quad x\ge 1\tag{3}$$.


Formula (3) above is equivalent to

$$b_o(x)=\lim\limits_{\epsilon\to 0} \left(\frac{b(x-\epsilon)+b(x+\epsilon)}{2}\right)\\=x\, (\log(x)-1)+1+\sum\limits_{\rho} \frac{x^{\rho} (1-\rho \log(x))-1}{\rho^2}\\+\sum\limits_{n=1}^\infty \frac{x^{-2 n} (1+2 n \log(x))-1}{4 n^2}\\=x\, (\log(x)-1)+1+\sum\limits_{\rho} \frac{x^{\rho} (1-\rho \log(x))-1}{\rho^2}+\frac{1}{4} \left(\text{Li}_2\left(\frac{1}{x^2}\right)-2 \log(x) \log\left(1-\frac{1}{x^2}\right)-\frac{\pi^2}{6}\right),\quad x\ge 1\tag{4}$$

which makes it more clear that $b_o(1)=0$.

$\endgroup$

You must log in to answer this question.

Start asking to get answers

Find the answer to your question by asking.

Ask question

Explore related questions

See similar questions with these tags.